Wealth Management Exams 1 & 2 Study Guide

अब Quizwiz के साथ अपने होमवर्क और परीक्षाओं को एस करें!

The use of which of the following risk measures was proposed by Markowitz as the appropriate statistic when developing portfolios?

Standard deviation.

An investor wants to evaluate funds 1, 2, and 3 and the index. The risk-free rate is 3%. Using the Treynor Ratio, which one provides the highest risk-adjusted return?

Fund 2.

The least likely risk of a venture capital investment is:

Funding risk.

Year 2's net income is closest to:

$1,000.

The preferred stock for Iron Heights pays an annual dividend of $6.00 while the firm's preferred shareholders require an 9.6% return. The value of this stock is closest to:

$62.50.

The return on equity is closest to:

13.4%.

An investor buys 1 XYZ November 60 call at $5. When XYZ increases to 70, the call is exercised and the stock is immediately sold. What is the result?

A profit of $500.

Which of the following mutual fund types will most likely have the highest total risk level?

A small-cap growth fund.

Examples of anchoring most likely include:

An investor holding an underperforming equity security until its price rises by 25%.

Parker believes that Ace Stock is poised to increase. Which of the following strategies could he make money with if the stock increases?

Buy a call on Ace. Sell a put on Ace.

Jordan is considering purchasing a CMO. Which of the following is not correct?

CMOs are not subject to default risk.

A type of preferred stock whose payments, when missed, must be paid prior to paying dividends to common stock is:

Cumulative preferred stock.

Eric purchased a 12-year, 7% coupon bond that is callable in three years. Which type of duration is the best for Eric to use to estimate price changes?

Effective duration.

A call option is available on the stock of Rally Corporation. The exercise price is $60. The option price is $7. Rally stock is currently selling for $70 per share. What is the "intrinsic value" of the option?

$10.

The inventory turnover at year's end is closest to:

9.01.

With regard to futures contracts, "margin" is:

A good faith deposit made at the time of the contract is bought or sold.

Investors want to protect against downside risk. Which of the following transactions creates a floor, such that losses are limited?

Buying a put option.

Donna invested in Fund A and Fund B, both of which are appropriate for her level of risk tolerance. Details for the two funds are below. Which of the following is correct?

Fund B is the superior fund because its return is always greater than or equal to the return for Fund A.

An investor wants to evaluate funds 1, 2, and 3 and the index. The risk-free rate is 3%. Using the Sharpe Ratio, which one provides the highest risk-adjusted return?

Index.

What amount of assets were invested in investment companies at the end of 2017?

Over $20 trillion.

Unique aspects of real estate investing as alternative investments most likely include:

The ability to generate regular income

Which of the following is (are) correct regarding average returns?

The geometric mean is equivalent to IRR. + The arithmetic mean is the average return for a series of returns and will always be greater than or equal to the geometric mean.

Which of the following is correct regarding the reasons an investor might consider investing in corporate bonds?

They offer higher security of principal than equities.

Doug buys Mayonnaise, Inc. stock for $112 using a margin account with a 50% initial margin and a 35% maintenance margin. Assuming the price of the stock drops to $56, how much would Doug need to pay to restore the equity in his account to the maintenance margin?

$19.60

Wiley sold short 100 shares of ACME stock at a price of $80 per share. While the position was open, a dividend of $2 per share was distributed. Wiley covered the short position at $50 per share. What is his gain, ignoring commissions and borrowing costs?

$2,800.

Jackson Inc. has EPS of $5.625 with a retention ratio of 60%. The annual growth rate in dividends is expected to be 6% and Jackson's shareholders require a return of 11%. The stock price is closest to:

$47.70.

The Charlie bond is a 3% coupon bond with semi-annual coupon payments that matures in 30 years. If the YTM for this bond is 6%, what is the value of the bond?

$584.87.

Parker buys ten zero-coupon bonds with a maturity of 20 years and a yield to maturity of 7% for a total of $2,584.19. Assume he buys the bonds on July 1st. How much interest will he have to report for tax purposes for the first year? Assume annual compounding for simplicity and round your answer to the nearest dollar.

$90.

Harrison owns a convertible bond with an 6% annual coupon and a $1,000 face value. It matures in 15 years and can be exchanged for 40 shares of Ford stock, which is trading at $24 per share. Similar nonconvertible bonds are priced to yield 6.5%. The value of the convertible bond is at least ?

$960.00.

Long positions and short positions in a futures contract have certain requirements for the investor regarding delivery of the underlying commodity. Which of the following accurately describes the requirement for long positions and short positions?

(Long) Take Delivery / (Short) Make Delivery

The Miracle Whip Fund (MWF) generated a return of 14.2% over the risk-free return of 3.2%. The relevant market premium was 10%. If MWF's beta is 1.3, then what is Jensen's alpha for the fund closest to?

+1.2

Which of the following terms are specified in a futures contract?

1, 3, and 4.

Horace bought 10,000 shares of LaLa at $40 per share. Two years later he sold the stock for $80 per share. LaLa declared and paid a dividend of $4 per share during the period Horace held the stock. What was Horace's holding period return?

110%.

ACE has EPS of $10.00 per share and has a retention ratio of 80%. Its dividend is expected to grow at a rate of 9%. If ACE stock is trading at $54.50, then the shareholder's required return is closest to:

13%.

Malik David purchased 100 shares of the Canadian Textile Fund for CAD 25 at the beginning of the year and was subject to a front-end load of 4.5%. The net asset value of the fund increased by 11% during the year while the fund's expense ratio was 1.2%. The annual return earned by David if he sells at year's end is closest to:

4.86%.

The debt to asset ratio is closest to:

54.7%.

Elmer currently owns a mutual fund with a correlation of .93 with the S&P 500 index. Elmer would like to diversify his portfolio by adding either the Wabbit Fund, which has a .48 correlation with the S&P 500 index and a standard deviation of 5%, or the Yosemite Fund, which has a .95 correlation with the S&P 500 index and a standard deviation of 12%. Which of the following is correct?

Adding the Wabbit Fund will provide greater diversification due to its lower R2 value.

A portfolio manager who uses mean-variance optimization to construct an optimal portfolio will satisfy which of the following?

All investors, regardless of their level of risk.

Brooke and Brisco each invested $1,000 into a mutual fund. Brooke invested her money in a fund that generated an 11% return with a 1% expense ratio, netting a 10% return. Brisco invested in a different fund that generated an 11% return with a 2% expense ratio, netting a 9% return. How much more will Brooke have accumulated compared to Brisco after 20 years?

Between 20% and 30%.

The dividend-payout ratio is equal to:

Dividends per share divided by earnings per share.

Solid Rock Fund invests in long-term high quality corporate fixed-income securities. Investors would most likely be attracted to this fund during times of:

Falling interest rates.

Which of the following statements concerning risk is not correct?

Financial risk is associated with the use of equity as part of a company's capital structure.

Which of the following statements is correctly associated with William Sharpe's extension of modern portfolio theory?

For a diversified portfolio, the only risk worth pricing is the systematic risk of a security.

The goal of active management is to construct risky portfolios with:

Higher Sharpe ratios than the relevant benchmark index.

Dylan Hope uses commodity futures contracts as part of his search for low correlations and diversification for his equity portfolio. Which of the following decisions would most likely be described as behavioral in nature?

Hope avoids tobacco stocks because his grandmother died of lung cancer.

Blue-chip stocks are most likely:

Issued by reliable companies that have the potential to perform well in any market.

George purchased a call option on an "index" of stocks. He is only permitted to exercise it on one specific date. He has a:

Long European option.

Which of the following statements regarding mortgage-backed securities and fixed-rate bonds is NOT correct?

MBSs are subject to interest rate risk while fixed-rate bonds are not.

Which of the following is most accurate regarding investment companies?

Open-end funds are capable of issuing shares and redeeming shares on a daily basis.

To profit from an expected increase in interest rates, a trader would:

Sell Treasury bond futures contracts.

Sam has a long position in one coffee futures contract that expires in June. To close her position, she should:

Sell one coffee contract that expires in June.

An order to buy or sell a certain quantity of a security at a specific price or better, but only after a specified price has been reached, is called a:

Stop-limit order.

A business that cuts and sells lumber can most likely reduce its short-term commodity price risk by

Taking the short position in a lumber futures contract.

All of the following statements regarding securities laws and regulation are correct EXCEPT:

The Securities Act of 1933 created the Securities and Exchange Commission (SEC).

Which of the following statements regarding plain vanilla interest rate swaps is NOT correct?

The full amount of interest received by each party is paid to the other.

Which of the following is the best reason to invest in alternative investments?

To achieve additional risk reduction through the addition of a low correlation asset.

Which of the following is not part of the Securities and Exchange Commission's mission?

To insure against large losses for issuers in the primary market.

Toby recently graduated from college with a degree in finance. Toby enjoys stock analysis and is eager to get started with investing in the market. He has approximately $30,000 of student loan debt at an average interest rate of 4.2%, and the $300 monthly payment is easily manageable even at the starting salary for a new college graduate. Although Toby does not currently have any savings, he has asked a financial planner to assist him with opening a brokerage account where he can begin buying stocks to save toward his goal of buying a condo in the next 7-8 years. Which of the following is true?

Toby's willingness (propensity) to take on risk is greater than his ability (capacity) to take on risk, so the planner should encourage him to accumulate emergency funds prior to purchasing stocks in a brokerage account to save for the goal.

Which of the following is correct regarding a long straddle?

Total premiums paid is the maximum loss.

A portfolio manager who earns a negative return to selectivity most likely has:

Invested in assets that are overvalued.

A futures contract:

Is an agreement to buy or sell a specified amount of an asset at a predetermined price on the expiration date of the contract.

Assume Marleen adds security Y to her portfolio that is less than perfectly positively correlated with the portfolio. Security Y has the same standard deviation as the portfolio. What will happen to the standard deviation of the portfolio after she adds security Y?

It will decrease.

Treasury bills are discount securities most likely because they:

Make no explicit interest payments.

Futures exchanges require all contracts to be for the purpose of minimizing default risk.

Marked-to-market.

A portfolio manager believes the price of AOC stock will fall over the next three months. The position most likely to result in a profit if the manager is accurate is:

Purchase a put option on AOC.

One part of the concept of Prospect Theory states that people place undue emphasis on low probability events that have large potential losses. Which of the following is the best example of this behavior mistake?

Purchasing insurance with excessive coverage at high premium cost.

A bond for which the bondholder has the right to cash in the bond before maturity at a specific price after a specific date is a:

Puttable bond.

Security X and Security Y have a correlation coefficient of 0.0. If Security X's price is expected to increase by 8%:

Security Y's return cannot be calculated.

According to William Sharpe's extension of modern portfolio theory, which of the following is the most relevant for securities pricing?

Systematic risk of a security in a diversified portfolio.

Relevant variables for computing the net asset value of a mutual fund most likely exclude:

The initial outstanding shares

The basis for a futures contract is defined as:

The spot price minus futures price.

What is a disadvantage of an ETF?

They can be subject to brokerage commissions.

An investor uses fundamental analysis to form a portfolio of equity securities. The portfolio has outperformed its benchmark for a period of almost a decade. The market under these conditions is most likely:

Weak form efficient.

Ollie, who is a financial advisor, manages an equity portfolio for the Green Arrow University endowment fund, which has an 8.1% return objective. Ollie makes a strategic allocation recommendation that produces a return of 7.8% in an economy that has experienced a 2.7% rate of inflation. Ollie also creates his own benchmark for the fund which includes multiple indexes that have similar risk profiles of the securities in the fund. The benchmark return during the period is 7.5%. Is the endowment fund satisfied with Ollie's performance?

Yes, because the fund outperformed the benchmark return.

The T-bill has a beta equal to ____, while the market portfolio's beta is equal to ____.

Zero; one.

Jack owns a convertible bond with a $1,000 face value that can be exchanged for 25 shares of WUF stock, which is trading at $50 per share. The conversion value equals?

$1,250.

An investor buys $100,000 in par value of TIPS with a coupon rate of 8%. Inflation during the first six months is 2.4%. The first semi-annual coupon payment is closest to:

$4,096.

Edgar Corp (EC) is a growth company that has never paid a dividend. The EC board of directors has decided to pay its first dividend one year from today. The first dividend will be $2.00 per share. Because of the growth expectations for the company, it is expected that the following two dividend payments will increase by 40% each year. Beyond that, the EC dividend is expected to grow at 6% annually. What is the value of EC if the required rate of return equals 11%?

$67.71.

Rodgers Industries pays a current dividend of $5.50 and shareholders require a 14% return. The dividend will grow at a high rate of 20% and then gradually decline to 4% over an eight-year period. The value of Rodgers Industries shares using the H Model is closest to:

$92.40.

What is the Jensen measure of performance for the Panda Fund? The risk-free rate for the period was 4%.

-1.2.

Which of the following factors should be considered when valuing a real estate investment? 1) After-tax cash flow, calculated as net after-tax earnings, plus depreciation minus principal repayment. 2) Net operating income divided by an appropriate capitalization rate. 3) The cost of the investment, which is calculated by adding maintenance costs, depreciation, and taxes to the purchase price, in comparison to other investment options.

1 and 2.

Which of the following statements concerning no-load mutual funds is (are) correct? 1. These funds do not charge a sales commission, but they may collect a redemption fee. 2. These funds may charge 12b-1 fees for marketing expenses. 3. Shares of these funds may be sold without a prospectus because they are sold by direct mail and media advertising.

1 and 2.

Which of the following statements concerning mutual funds are correct? 1. They provide diversification for investors who have only a small amount to invest. 2. From the initial purchase of mutual fund shares, an investor is assured of a minimum income level and a stated redemption price. 3. They do not charge sales fees on initial investments. 4. Their earnings are exempt from federal income taxes at the corporate (mutual fund) level.

1 and 4.

The sustainable growth rate for Year 2 is closest to:

1.25%.

Bud is considering purchasing an 8-year bond that is selling for $700. What is the YTM for this bond if it has a 6% coupon, paid semi-annually?

11.92%.

The gross profit margin on sales is closest to:

20.0%.

The return on equity using the DuPont system is closest to:

21.76%.

Which of the following bonds will have the largest decrease in price if interest rates increase in Year 1 of the life of the bonds?

A 10-year zero coupon bond.

Private equity firms are least likely known for:

Acting as limited partners in the business relationship.

A certificate issued by U.S. banks representing ownership in shares of a stock of a foreign company that are held on deposit in a bank in the firm's home country is:

American Depositary Receipts (ADRs).

Which of the following statements about Treasury bills and commercial paper is correct?

BOTH OF THE FOLLOWING ARE FALSE: T-bills and commercial paper are default-risk free. Yields on T-bills are slightly higher than commercial paper since they are issued by the United States government.

JoJo enjoyed collecting baseball cards with her father when she was a child, and as an adult has an extensive collection included in her investment portfolio. Which of the following pose significant risk for JoJo in regard to her baseball card collection?

Behavior biases. Changes in consumer preferences.

Sophia is considering an investment in soft commodities to add diversification to her portfolio. All of the following are likely to have a significant impact on Sophia's return EXCEPT:

Changes in U.S-CAD exchange rates.

The authority function of a self-regulatory organization is most likely characterized by:

Creation and enforcement of its own policies.

The Jensen Hybrid Fund would most likely be chosen as an investment vehicle by:

Investors seeking to plan for retirement.

An "out of the money put:"

Is where the stock price > strike price.

Which of the following is correct?

STRIPS are subject to the OID rules

Which of the following statements about Treasury bills is correct?

T-bills are sold on a pure discount basis only.

A split bond rating would most likely occur when:

Two ratings agencies disagree on the default level of an issue.

Beta should be used to measure which of the following?

Undiversifiable risk.

Hayden bought 15 ABC June 60 calls at $7 and bought 15 ABC June 60 puts at $6.10. The maximum possible gain on this combination of positions is:

Unlimited.

The net gain on buying a put option for $10 when the stock price is $45, the exercise price is $25, and the expiration date stock price is $20 is closest to:

($5).

What is the impact of an increase in basis to a long hedger and to a short hedger?

(Long) Gain / (Short) Loss

A firm with a price-earnings ratio of 8.39 has earnings per share of $5.38. This firm will have an expected stock price closest to:

$45.14.

Jacques bought a bond with a modified duration of 4.90. By approximately what percentage will the bond price change assuming interest rates decrease by 110 basis points?

+5.39%.

Beth, who lives in NY City, is in the 24% federal tax bracket and 6% state income tax bracket. Which of the following bonds that she is considering purchasing has the highest after-tax yield? 1) Treasury bond paying 5.4% 2) Corporate bond paying 5.5% 3) Florida Municipal bond paying 4.2%

1 only.

Portfolio D has a standard deviation of 17% and a correlation with the market of 1.00. If the standard deviation of the market is 15%, what is the beta for D?

1.13.

The current ratio is closest to:

1.68.

Assume that the three-month return for Hunter stock is 3%. What is the annualized return for Hunter?

12.55%

Assume that the 1-year rate is 7% and the 2-year rate is 10%. Based on the expectations theory, what is the implied 1-year rate, 1 year from today?

13%.

The expected market return is 12% and the risk-free rate of return is 3%. Using the CAPM (SML), what is the expected return for a portfolio that has a standard deviation of 18% and a beta of 1.3?

14.7%

Allison has owned an interest in a company called John Enterprises (JE). During the same time, Treasuries have yielded a 3% return. JE has an average return of -3% with a standard deviation of 6%. Assuming the returns are normally distributed, what is the probability that JE will have a return greater than the Treasury securities?

16%.

Colin, a citizen of the USA, invested $1.5 million in Fish N Chips, a UK company, trading at a market value of £35 per share. The conversion rate for pounds to dollars was £1 to $1.70 at the time of the investment. Assume that after two years, he sells the stock for £40 per share when the conversion rate for pounds to dollars is £1 to $1.8. How much is his gain?

21.00%.

Nyssa is considering two portfolios: 1) Portfolio A with a return of 11% and a standard deviation of 16% and 2) Portfolio B with a return of 6% and a standard deviation of 8%. Assuming the correlation between A and B is -0.3, what of the following is the most efficient portfolio?

30%A/70%B.

Donna is considering purchasing a 3-year bond that is selling for $1,000. What is the current yield for this bond if it has a 4% coupon, paid semi-annually?

4%.

Which of the following is closest to the number of stocks in the Wilshire 5000 Total Market Index?

500.

The Yoko Fund earns 11.2% during the year while the risk-free rate is 3.1%. The Yoko Fund has a beta of 1.20 and a standard deviation of 17.5%. The Treynor Ratio is closest to:

6.750

A firm in the water bottling industry pays a current dividend of $2.19. Its earnings per share is $8.36 and an analysis of the financial statements shows a return on equity of 9.32%. The sustainable growth rate is closest to:

6.88%.

Parker invested $3,000 in CJD stock, $3,000 in JCD stock, and $3,000 in JFD stock. CJD, JCD, and JFD have expected returns of 6%, 7% and 10%, respectively. What is the weighted average expected return for Parker's portfolio?

7.67%.

Eunice is single and in the 24% federal and 5% state tax brackets. She is considering the purchase of a municipal bond, issued in her state of residence, with a YTM of 6.75%. What is Eunice's tax equivalent yield on the bond?

9.51%

Jordan has a portfolio of mutual funds, A, B, and C. She has 50% in A, 40% in B, and 10% in C. What is the expected return for the portfolio if the relative expected returns for A, B, and C are 10%, 8%, and 14%?

9.60%.

Nate invested in the Not-so-Good mutual fund five years ago. His returns were -5%, -8%, -5%, -6% and -8%, respectively. What is the difference between the arithmetic average and the geometric average return over the five years?

<0.01%.

What is a bond issued and supported only by the general credit standing of the issuing corporation called:

A debenture.

Firms are least likely to use the primary equity market to raise capital for:

A desire to increase its financial leverage.

Which type of underwriting arrangement is the riskiest to the underwriter?

A firm commitment.

An investor subject to behavioral biases is most likely to:

Allow emotions to determine buy/sell decisions.

A portfolio manager generates a return of 14.9%. The relevant benchmark return is 13%. The contribution of asset selection is -40 basis points. Which of the following is most accurate?

Asset allocation contributes 230 basis points to total performance.

Emma Jones shows signs of being averse to losses. Jones will most likely:

Avoid selling stocks that would generate capital loss.

Which of the following statements is true about equity returns, as represented by the S&P 500 from 1928 - 2016?

BOTH BELOW ARE FALSE: The historical 1-year returns for the S&P 500 have ranged from approximately 25% to -15%. The lowest 20-year average return for this time period was -3.0%.

Guinness is considering two portfolios: 1) Portfolio A with a return of 14% and a standard deviation of 14% and 2) Portfolio B with a return of 4% and a standard deviation of 7%. Assuming the correlation between A and B is 0.5 and he invests 70% in A and 30% in B, what range of returns should this portfolio produce 95% of the time?

Between -11% and 33%.

Parker has a portfolio with a beta of 1.0 and an alpha of 0. Based on the CAPM, the return for the portfolio is __________.

Equal to the Rm.

According to Porter's 5 competitive forces used in industry analysis, rivalry among companies in the same industry tends to increase under which of the following conditions?

Firms in capital intensive industries characterized by high fixed costs produce high outputs when demand is low.

All of the following are relative valuation models, except:

H model.

Which of the following statements regarding technical and fundamental analysis is true?

In fundamental analysis, future cash flows are projected by analyzing a firm's financial statements.

Warren Buffet famously said, "You only find out who is swimming naked when the tide goes out." The quote, in part, means that when the equity market is increasing, it hides many weaknesses of businesses. What type of risk is this quote referring to?

Market risk.

Consider the graph with the SML. Which of the following statements is correct?

NONE OF THE BELOW ARE TRUE: The beta for Portfolios A, B, and C are all the same as they are all on the SML. The return for Portfolio C will be equal to the market premium minus the risk-free rate. If the return for the market was 12% and the risk-free rate was 3%, then A's expected return could not equal 15%.

Which of the following is correct about the application of duration?

NONE OF THE FOLLOWING ARE CORRECT: Duration can be used to provide an exact price impact from changes in interest rates. Yield curve risk does not impact duration. Effective duration is another name for modified duration.

Which of the following statements is correct?

NONE OF THE FOLLOWING ARE CORRECT: The coupon rate is the rate of interest paid on the market value of a bond. The YTM for a bond is the IRR of the cash flows that the investor earns if the bond is held to maturity. Callable bonds tend to have a lower YTM than non-callable bonds with the same default risk and maturity.

The management effect in fixed income attribution analysis least likely includes:

Policy Effect.

Consider the chart with the six portfolios and the efficient frontier. Which of the following portfolios is the most efficient in terms of risk and return?

Portfolio D (The farthest to the right that's on the efficient frontier curve).

A mortgage-backed security can be characterized as having:

Prepayment risk.

Generally, fixed-income investors are subject to which of the following risks?

Purchasing power risk. Interest rate risk. Default risk.

A money manager generates a return of 12% when the benchmark returns 10%. The manager's asset allocation decisions underperform by 25 basis points. Which is most accurate?

Security selection decisions outperform by 225 basis points.

Dinah's portfolio consists of a 50% equity index fund and a 50% fixed income index fund. The portfolio will be reallocated using the constant weighting method. More details are shown below: Dinah should do which of the following if the price of the Equity Fund changes to $20.00 and the price of the Fixed Income Fund changes to $30.40?

Sell 50 shares of Equity and purchase 33 shares of Fixed Income.

ZAGG investments owns 285,000 shares of CRC, a defensive stock. After examining the stock's cash flows, its executive leadership, and its likelihood of becoming a takeover target, a research analyst estimates the intrinsic value for this firm to be $35.00. The current market price on the NASDAQ exchange is $59.23. The analyst is most likely to recommend:

Selling the shares that are already owned.

Assume that the shareholders of a company anticipate receiving a normal dividend. What benefit would shareholders receive if the company instead repurchased shares with the same total amount of dollars that would have been spent on dividends? Assume that the PE ratio is maintained under either scenario.

Shareholders can delay or reduce taxes.

All of the following are correct regarding private placements except:

The downside to private placement is that it is expensive and time consuming.

Which of the following is not correct regarding the constant growth dividend discount model?

The model requires that the required return be greater than or equal to the growth rate of the dividend.

A fixed-income portfolio manager with excellent access to an efficient trading desk and with a reliable record of making profitable short-term substitutions will most likely outperform the benchmark due to:

Trading effect.

Chris Cowlings, Chartered Financial Analyst®, uses fundamental analysis to evaluate equity securities in the country of Gooseburg. Cowling has been able to use inflation data, historical dividend information, and price to book rations to consistently outperform Gooseburg's broad equity index. Cowling would most likely identify the markets in Gooseburg as being:

Weak form.


संबंधित स्टडी सेट्स

Diary of Anne Frank Play: Act 2, Scene 4-5

View Set

World History Study Guide 4-1, 4-2, and the nile article

View Set

Fa davis : Obstructive Pulmonary Disorders

View Set

California: Real Estate Principals Review

View Set

Test #1, Steve Shaner, Advertising

View Set